JEE Main & Advanced Mathematics Trigonometric Identities Question Bank Trigonometrical ratios of sum and difference of two and three angles

  • question_answer
    If \[\tan \alpha =\frac{m}{m+1}\]and \[\tan \beta =\frac{1}{2m+1}\], then \[\alpha +\beta =\] [IIT 1978; EAMCET 1992; Roorkee 1998; JMI EEE 2001]

    A) \[\frac{\pi }{3}\]

    B) \[\frac{\pi }{4}\]

    C) \[\frac{\pi }{6}\]

    D) None of these

    Correct Answer: B

    Solution :

    We have, \[\tan \,\alpha =\frac{m}{m+1}\] and \[\tan \,\beta =\frac{1}{2m+1}\] We know \[\tan \,(\alpha +\beta )=\frac{\tan \,\alpha +\tan \,\beta }{1-\tan \,\alpha \,\tan \,\beta }\] \[=\frac{\frac{m}{m+1}+\frac{1}{2m+1}}{1-\frac{m}{(m+1)}\,\frac{1}{(2m+1)}}=\frac{2{{m}^{2}}+m+m+1}{2{{m}^{2}}+m+2m+1-m}\] \[=\frac{2{{m}^{2}}+2m+1}{2{{m}^{2}}+2m+1}=1\,\,\Rightarrow \,\,\tan \,(\alpha +\beta )=\tan \frac{\pi }{4}\] Hence, \[\alpha +\beta =\frac{\pi }{4}\]. Trick : As \[\alpha +\beta \] is independent of m, therefore put \[m=1,\] then \[\tan \,\alpha =\frac{1}{2}\] and \[\tan \,\beta =\frac{1}{3}\]. Therefore, \[\tan \,(\alpha +\beta )=\frac{(1/2)+(1/3)}{1-(1/6)}=1.\] Hence \[\alpha +\beta =\frac{\pi }{4}.\] (Also check for other values of m).


You need to login to perform this action.
You will be redirected in 3 sec spinner